LSAT 91 – Section 2 – Question 25

You need a full course to see this video. Enroll now and get started in less than a minute.

Target time: 1:41

This is question data from the 7Sage LSAT Scorer. You can score your LSATs, track your results, and analyze your performance with pretty charts and vital statistics - all with a Free Account ← sign up in less than 10 seconds

Question
QuickView
Type Tags Answer
Choices
Curve Question
Difficulty
Psg/Game/S
Difficulty
Explanation
PT91 S2 Q25
+LR
PSA - Find or complete the application +PSAa
A
23%
158
B
4%
153
C
7%
154
D
7%
153
E
58%
164
148
158
168
+Harder 145.724 +SubsectionMedium

The question stem says of the following judgments, which one most closely conforms to the principle above? This is a rarer type of question though we have seen it plenty before. They're asking us to take the principle in the stimulus which is a conditional statement and push it into the arguments in the answers to see where it fits. But that’s like a PSA question. Instead of the stimulus containing an argument searching for a conditional in the answer, it's the other way around. The stimulus contains a conditional searching for an argument. This is a cosmetic difference.

The stimulus lays down two jointly sufficient conditions for justified governmental interference with an individual’s actions. The two conditions are:

  1. The action would increase the likelihood of harm to others; and
  2. The action is not motivated by a desire to help others.

If both conditions are met, then the government is justified in interfering with the individual’s action.

As an aside, note that these two conditions each cover a different kind of consideration. The first looks at the consequences of the action. Will this action harm others? The second looks at the intent of the action. What motivated the action? In general, considerations of morality tend to fall into these two buckets of consequences and intent.

Back to the task at hand. Given that this is a PSA question, it’s important to note which conclusions are reachable and which are unreachable.

A reachable conclusion is that the government is justified in interfering with the individual’s action. To reach this conclusion, we just have to show that (1) and (2) are satisfied.

An unreachable conclusion is that the government is not justified in interfering with the individual’s action. There’s simply nothing we can show to trigger the conditional in that way. If we wanted to reach the unjustified conclusion, we need to know what the necessary conditions of justified are, fail those conditions, then contrapose back.

This analysis is helpful in eliminating Answer Choice (A). It tries to conclude that the government is unjustified in interfering with Jerry’s moviemaking. We don’t need to read the rest of the argument. There’s nothing that the premise can state that will make use of the conditional to reach this conclusion. (A) is therefore wrong on its logic alone. In other words, it makes a sufficiency-necessity confusion, the oldest mistake in the book. We can stop here, but for review, look at the premise. It says that Jerry’s action (moviemaking) doesn’t harm and won’t increase the likelihood of harming anyone. It also says that it is motivated by a desire to help others. So it fails both (1) and (2). But failing sufficient conditions just makes the rule go away. It doesn’t trigger anything. Yet, (A) thinks it triggers the failure of the necessary condition. That’s textbook sufficiency-necessity confusion.

Contrast this with Correct Answer Choice (E). It concludes that the government is justified in preventing Jill from giving her speech. That’s a reachable conclusion. We just need to show that Jill’s speech satisfies (1) and (2). And it does. Her speech “would most likely have caused a riot and people would have gotten hurt.” That’s physical harm to others. And her speech was “to further her own political ambitions.” That’s a selfish motivation and hence not a motivation to help others.

Answer Choice (B) concludes that the government is justified in fining the neighbor for not mowing his lawn. That’s a reachable conclusion. We just need to show that the neighbor’s not mowing his lawn satisfies (1) and (2). (1) is problematic. It’s not pleasant to look at an unkempt lawn, but that’s not physical harm to others. We don’t need to consider (2) but probably the neighbor’s decision to not mow his lawn was selfishly motivated. He was probably just feeling lazy.

Answer Choice (C) concludes that the government is justified in requiring motorcyclists to wear helmets. That’s a reachable conclusion. We just need to show that motorcyclists’ not wearing helmets satisfies (1) and (2). Again, we have a problem for (1). It’s not clear that their failure to wear helmets would increase harm to others, whatever the consequences of harm are for themselves. We don’t need to consider (2) but probably their decision to not to wear helmets was selfishly motivated. They probably were feeling lazy, wanted to look cool, or have a death wish.

Answer Choice (D) concludes that the government is justified in suspending Z’s license to test new drugs. That’s a reachable conclusion. We just need to show that Z’s testing new drugs satisfies (1) and (2). Again, we have a problem for (1). It’s not clear that their testing of new drugs would increase harm to others. In fact, if they’re a drug company, then it’s more likely that their testing of new drugs would do just the opposite. It would help others alleviate pain and suffering. We don’t need to consider (2) but here the argument makes explicit that their motivation is selfish and not to help others.

Take PrepTest

Review Results

Leave a Reply